Đến nội dung

Hình ảnh

Cho $a,b,c>0$ Tìm min $P=\sum_{ciclic}^{.}\frac{(a+b-c)^2}{a^2+b^2+c^2+2ab}$


  • Please log in to reply
Chủ đề này có 5 trả lời

#1
nbngoc95

nbngoc95

    Trung sĩ

  • Thành viên
  • 106 Bài viết

Cho $a,b,c>0$ Tìm min $P=\sum_{ciclic}^{.}\frac{(a+b-c)^2}{a^2+b^2+c^2+2ab}$



#2
banhgaongonngon

banhgaongonngon

    Thượng úy

  • Thành viên
  • 1046 Bài viết

Cho $a,b,c>0$ Tìm min $P=\sum_{ciclic}^{.}\frac{(a+b-c)^2}{a^2+b^2+c^2+2ab}$

 

$\sum \frac{(b+c-a)^{2}}{a^{2}+(b+c)^{2}} \geq \frac {3}{5}$

Chuẩn hóa $a+b+c=3$

Ta có $\sum \frac{(3-2a)^{2}}{a^{2}+(3-a)^{2}}\geq \frac{3}{5}$

$\Leftrightarrow \sum \frac{4a^{2}-12a+9}{2a^{2}-6a+9}\geq \frac{3}{5}$

Mặt khác, ta có $\frac{4a^{2}-12a+9}{2a^{2}-6a+9}\geq \frac{1}{5}-\frac{18}{25}(a-1)$

Lập $2$ bất đẳng thức tương tự, ta có đpcm



#3
nbngoc95

nbngoc95

    Trung sĩ

  • Thành viên
  • 106 Bài viết

$\sum \frac{(b+c-a)^{2}}{a^{2}+(b+c)^{2}} \geq \frac {3}{5}$

Chuẩn hóa $a+b+c=3$

Ta có $\sum \frac{(3-2a)^{2}}{a^{2}+(3-a)^{2}}\geq \frac{3}{5}$

$\Leftrightarrow \sum \frac{4a^{2}-12a+9}{2a^{2}-6a+9}\geq \frac{3}{5}$

Mặt khác, ta có $\frac{4a^{2}-12a+9}{2a^{2}-6a+9}\geq \frac{1}{5}-\frac{18}{25}(a-1)$

Lập $2$ bất đẳng thức tương tự, ta có đpcm

Mình ko hiểu chỗ 

$\sum \frac{(b+c-a)^{2}}{a^{2}+(b+c)^{2}} \geq \frac {3}{5}$

Chuẩn hóa $a+b+c=3$



#4
kobietlamtoan

kobietlamtoan

    Trung sĩ

  • Thành viên
  • 112 Bài viết

Mình ko hiểu chỗ 

$\sum \frac{(b+c-a)^{2}}{a^{2}+(b+c)^{2}} \geq \frac {3}{5}$

Chuẩn hóa $a+b+c=3$

đó là bạn thay a = b = c vào và dự đoán Min đó mà.


Nghiêm Văn Chiến 97

#5
nbngoc95

nbngoc95

    Trung sĩ

  • Thành viên
  • 106 Bài viết

Mặt khác, ta có $\frac{4a^{2}-12a+9}{2a^{2}-6a+9}\geq \frac{1}{5}-\frac{18}{25}(a-1)$

 

Còn đoạn này? Sao lại $\geq \frac{1}{5}-\frac{18}{25}(a-1)$



#6
kobietlamtoan

kobietlamtoan

    Trung sĩ

  • Thành viên
  • 112 Bài viết

Còn đoạn này? Sao lại $\geq \frac{1}{5}-\frac{18}{25}(a-1)$

cái này bạn k hiểu cũng đúng thôi. bạn ấy làm tắt mà. bạn k hiểu vì sao là $\geq \frac{1}{5} - \frac{18}{25}(a-1)$

đơn giản là như thế này. Ta cần chứng mình: $\sum \frac{4a^2-12a +9}{2a^2 - 6a+9} \geq \frac{3}{5}$ 

do a+b+c = 3 nên (a-1) + b-1 +c-1 = 0. do đó ta sẽ có hướng làm:

$\frac{4a^2-12a +9}{2a^2 - 6a+9} \geq \frac{1}{5} + k(a-1)$

tương tự vs 3 cái kia rồi cộng vô được đpcm.

Vậy ta cần tìm hệ số k để bất đẳng thức thỏa mãn.

Ta có:  

$\frac{4a^2-12a +9}{2a^2 - 6a+9}- \frac{1}{5}- k(a-1) = \frac{ 18a^2-54a+36}{2a^2-6a + 9} - k(a-1) = \frac{18(a-1)(a-2)}{2a^2-6a+9} - k(a-1)$

 

$\frac{18(a-1)(a-2)}{2a^2-6a+9} - k(a-1) = (a-1)(\frac{18a-36}{2a^2-6a + 9}-k)$

 

Cho a = 1 vào để có : $\frac{18a-36}{2a^2-6a + 9}-k =0$ với a = 1 Tìm đc k. đến lúc này bạn tự tính xem bđt đúng vs k = ngần ấy chưa


Nghiêm Văn Chiến 97




1 người đang xem chủ đề

0 thành viên, 1 khách, 0 thành viên ẩn danh